BOWLING The cost for Nobu to go bowling is $4 per game plus an additional flat fee of $3.50 for the rental of bowling shoes. The cost can be modeled by the function f(x)=4x+3.5, where x represents the number of games bowled. Describe the graph of g(x) as it relates to f(x) if Nobu does not rent bowling shoes.



g(x) = , 1 of 3.
Select Choice
, which is the translation of f(x) , 2 of 3.
Select Choice
units , 3 of 3.
Select Choice

Answers

Answer 1

The graph of g(x) = 4x, is the translation of f(x) by subtracting 3.5 units, 2 of 3.

What is a graph?

A graph can be defined as a pictorial representation or a diagram that represents data or values.

The cost can be modeled by the function f(x)=4x+3.5, where x represents the number of games bowled.

As per the given question, the required solution would be as:

g(x) = 4x , is the translation of f(x) by subtracting 3.5 units, 2 of 3.

The graph of g(x) would be identical to the graph of f(x), but shifted 3.5 units downward, 3 of 3.

This is because the flat fee for the rental of bowling shoes is not present in the function g(x), so the y-intercept of the graph would be at (0,-3.5) instead of (0,0) in f(x) function.

The slope and the shape of the function will remain the same.

Learn more about the graphs here:

brainly.com/question/16608196

#SPJ1


Related Questions

Complete the following indirect proof (proof by contradiction).
Given: Adjacent angles LA and ZB, formed by the intersection of two lines
Prove: At least one of the angles LA and B has measure 90° or greater

Answers

First, we assume that this conclusion is false. In other words, we assume that the contrary statement "none of the two angles has measure [tex]90^{\circ}[/tex] or greater" is true.

The assumption is equivalent to the following two statements:

(1) [tex]m\angle A\text{ } \boxed{ < 90^{\circ}}[/tex]

(2) [tex]m\angle B\text{ } \boxed{ < 90^{\circ}}[/tex]

Using (1) and (2) and the addition properties of inequalities, we conclude that [tex]m\angle A+m\angle B \text{ } \boxed{ < } \text{ } 180^{\circ}[/tex].

On the other hand, two adjacent angles form a linear pair. Thus, the last statement contradicts the Linear Pair Property, which states that for a linear pair of angles [tex]\angle A[/tex] and [tex]\angle B[/tex], [tex]m\angle A+m\angle B \text{ } \boxed{=} \text{ } 180^{\circ}[/tex].

Therefore, the assumption made is false, and the statement "at least one of the angles [tex]\angle A[/tex] and [tex]\angle B[/tex] has measure [tex]90^{\circ}[/tex] or greater" is true.

Please help me with this question

Answers

Therefore , the solution of the given problem of trigonometry comes out to be Cos 2x = -0.760355.

Trigonometry is what?

The study of relationships among triangle angles and angles is known as trigonometry. Mathematical modeling is a fundamental idea in all of geometry because it can be used to reduce any straight-sided form to a collection of triangles. Amazingly complex connections exist between trigonometry and other areas of mathematics, particularly mathematics, real or complicated, infinite series, logarithms, and infinite series.

Here,

Given : Cosx = 9/26 (First quadrant)

To find : Cos2x

We use the formula:

=>Cos2x = 2Cos²x -1

=> Cos2x  = 2(9/26)² - 1

=> Cos 2x = -0.760355

Therefore , the solution of the given problem of trigonometry comes out to be Cos 2x = -0.760355.

To know more about trigonometry , visit

brainly.com/question/12068045

#SPJ1

Find the sum and express it in simplest form.
(7t³ + 3t - 1) + (-9t³ + 2t)
DO
Enter the correct answer.

Answers

Answer:

-2[tex]t^{3}[/tex] + 5t - 1

Step-by-step explanation:

(7[tex]t^{3}[/tex] + 3t - 1) + (-9[tex]t^{3}[/tex] + 2t)

7[tex]t^{3}[/tex] - 9[tex]t^{3}[/tex] + 3t + 2t - 1  Combine like terms

-2[tex]t^{3}[/tex] + 5t - 1

Like terms have to have the same variable and the same exponents to be like terms.

PLEASE ANSWER

Write an equation in point-slope form passing through the given point and parallel to the given line:

(- 4, 4)

y = - 7/4x + 2

Answers

Answer:

y-4=-7/4(x+4)

Step-by-step explanation:

Point Slope Form: y-y*sub*1=m(x-x*sub*1)

Parallel lines have the same slope as the line to which they are parallel.

y-4=-7/4(x+4)

Hope this helps :)

Lorraine prepared a 2.5-gallon pot filled with tomatoes to be canned in jars. Each jar will hold 1.25 quarts of tomatoes. If 1 gallon equals 4 quarts, how many jars can Lorraine fill?

4 jars
5 jars
8 jars
10 jars

Answers

Answer:

  (c)  8 jars

Step-by-step explanation:

You want to know the number of 1.25 quart jars that can be filled from a 2.5 gallon container.

Jars

The number of jars can be found from ...

  (2.5 gal) × (4 qt/gal) × (1 jar)/(1.25 qt) = (2.5·4/1.25) jars = 8 jars

Lorraine can fill 8 jars from the 2.5 gallon pot.

<95141404393>

A community theater performed a play each Saturday evening for 10 consecutive
weeks. The graph shows the attendance for the performances in weeks 1, 4, 6, and 10.
Describe the rates of change in attendance with respect to time.
Weeks 1-4:
Weeks 4-6:
Weeks 6-10:
300
250
200
150
100
50
y
Attendance
Play Attendance
(4, 230)-
(1,131)
(6,208)
(10,60)
Week of performance
0
0 1 2 3 4 5 6 7 8 9101112

Answers

The rates of change in attendance in week 1-4 is 33, week 4-6 is -11, week 6-10 is -37 people per week

What is rate of change of attendance?

The rate of change in attendance with respect to time is calculated by taking the difference in attendance between two weeks and dividing by the number of weeks between those weeks. This is done to find the average rate of change in attendance per week. The formula used is:

[tex]Rate =\frac{Attendance at week 2 - Attendance at week 1}{week 2 - week 1}[/tex]

Weeks 1-4:

The rate of change in attendance with respect to time is 33 people per week. This can be calculated by taking the difference in attendance between weeks 1 and 4 (230-131) and dividing by the number of weeks between those weeks (4-1 = 3).

[tex]rate =\frac{230-131}{4-1}\\[/tex]

hence rate is 33 people per week

Weeks 4-6:

The difference in attendance between weeks 4 and 6 (230 - 208)

The number of weeks between those weeks (6-4 = 2).

[tex]rate =\frac{208-230}{6-4}[/tex]

Hence rate is -11 people per week

Weeks 6-10:

The difference in attendance between weeks 6 and 10 (60-208)

The number of weeks between those weeks (10-6 = 4).

[tex]rate=\frac{60-208}{10-6}[/tex]

Hence rate is -37 people per week

To know more about rate of change, check out:

https://brainly.com/question/12752108

#SPJ1

Assume that AGHIE ALMN. Which of the following congruence statements
are correct? Check all that apply.
A. ZL 21
B. IG LM
C. THNM
D. ZN 21
E. GH = LM
F. ZME ZH

Answers

On solving the provided questions, we can say that the congruence of ABC=DEF, where A=D, B=E, and C=F.D because they are equal in length. F

What is congruence?

A figure that can be put exactly on another is said to be "matching." When the bread is piled, it is the same size and shape. The term "match" describes things that are precisely the same shape and size. A pair of geometric figures is said to be congruent or to be in a congruent relationship if one of them can be reliably superimposed over the other.

B D and F.

B because they would go together if the statement were put in that particular sequence. ABC=DEF, where A=D, B=E, and C=F.D because they are equal in length. F due to the same cause as B.

To know more about  congruence visit:

brainly.com/question/7888063

#SPJ1

Solve for 0. Round to the nearest tenth of a degree. Sin 0=0.85

Answers

The value of the angle theta is 58 degrees

How to determine the value of the angle theta

From the question, we have the following parameters that can be used in our computation:

Sin 0=0.85

Express the angle in the equation properly

So, we have the following representation

sin(Ф) = 0.85

Take the arc sin of both sides of the equation

So, we have the following representation

sin⁻¹(sin(Ф)) = sin⁻¹(0.85)

Evaluate the arc sin of one side of the equation

So, we have the following representation

Ф = sin⁻¹(0.85)

Evaluate the arc sin of the other side of the equation

So, we have the following representation

Ф = 58.2116693829

Approximate the equation

Ф = 58 degrees

Hence, the solution is 58 degrees

Read more about trigonometry at

https://brainly.com/question/24349828

#SPJ1

name all the terms realted to subtraction ​

Answers

Answer:

Other words for subtraction:

taking away

withdrawal

abstraction

removal

discounting

Step-by-step explanation:

4. Of the three elevations, -2 feet, -12 feet, and
30 feet, which represents the least number?
Which represents the farthest distance from
sea level?

Answers

On solving the provided question, we can say that - y proportional to  as the cube of s, y = ks3, then  8 = k * 23 => k = 1 then y  = 1 * 53 = 125

what is directly proportional?

If two numbers are directly proportional, then as one quantity grows by a specific percentage, so does the other quantity by the same percentage. As an illustration, as gas costs rise, the price of food may also increase. If the ratio of two values x to y never changes, then x and y are said to be directly proportional to one another. example: Let's say you spend Rs. 50 on two pens. 4 pens cost 100 rupees each. If the ratio of two numbers, x and y, is greater than 1, then the two values The value of y1 is constant.

Since y proportional to  as the cube of s, y = ks3.

8 = k * 23 => k = 1

then y  = 1 * 53 = 125

To know more about directly proportional visit:

brainly.com/question/2548537

#SPJ1

To make an apron, Jaylen’s mother bought 3.9 yards of cloth. If a yard of cloth costs $9.50, how much did Jaylen’s mother spend?

Answers

Answer:

She spent 37.05

Step-by-step explanation:

multiply both values

The perimeter of a rectangle is 64 feet. The length is two more than double the
width. Find the dimensions of the rectangle.

(Remember Perimeter of rectangle = 2L+ 2w)

Answers

Answer:

To solve this problem, we can set up the following equation:

2L + 2W = 64

We are told that the length of the rectangle is two more than double the width, so we can write the following equation:

L = 2W + 2

Substituting this equation into the first equation, we get:

2(2W + 2) + 2W = 64

Simplifying this equation, we get:

6W + 4 = 64

Subtracting 4 from both sides, we get:

6W = 60

Dividing both sides by 6, we get:

W = 10

The width of the rectangle is 10 feet.

To find the length of the rectangle, we can substitute this value back into the equation L = 2W + 2 to get:

L = 2(10) + 2

= 20 + 2

= 22

The length of the rectangle is 22 feet.

Therefore, the dimensions of the rectangle are 10 feet by 22 feet.

Step-by-step explanation:

Width = 10
Length = 22

Explanation

1. Write the equation L= 2w + 2 for the given statement length = two more than doubled word.

2. 2(2w+2) represents 2L in the perimeter equation. Solve 2(2w+2) + 2w = 64 to find the value of the width.

3. Distribute the 2 into the parentheses first, which should give you 4w + 4 + 2w = 64.

4. Combine like terms 4w and 2w leaving you 6w + 4 = 64.

5. Subtract 4 on both sides of the equation to get 6w alone. 6w = 60.

6. Divide by 6 on both sides to solve for w. w = 10.

7. Input x = 10 into 2w + 2 to find the length of the rectangle. 2(10) + 2 = l

8, multiply 2 by 10, which is 20. Add that to 2, which is 22.

If the HCF is 5 and the product of the two numbers is 70, find the value of LCM.

Answers

If the HCF is 5 and the product of the two numbers is 70, then the value of LCM is 14.

Do two numbers make a product when they are the LCM of two numbers?

The smallest number that can be divided by both numbers is known as the least common multiple, or LCM, of two numbers. The product of the two numbers is the LCM if their greatest common divisor is 1, in which case their product is 1.

Using the fundamental theorem of mathematics, how do you find the HCF and LCM?

First, multiply both numbers by their prime factors to express them. Then, multiply the common factor with the highest power by the remaining factors to obtain the LCM. Take the common factors with the lowest power and multiply them to determine HCF.

To find the LCM of two numbers, you can use the formula: LCM = (a * b) / HCF.

In this case, we know that the HCF is 5, and the product of the two numbers is 70. Therefore, we can substitute these values into the formula:

LCM = (70) / 5 = 14.

So the LCM of the two numbers is 14.

To know more about HCF visit:-

brainly.com/question/29114178

#SPJ1

Find the area of the shapes

Answers

Answer:

45.5km

Step-by-step explanation:

AREA= base x height

So with this equation, we know that 5km is the height, and 9.1km is the base.

So to find out the area all we have to do is 9.1x5 which equals 45.5


Before the trip, Tyler read 24 pages of his Philadelphia guidebook in an hour. The guidebook is 84 pages long. If he continued to read it at the same
rate, how long did it take Tyler to finish reading the guidebook before his family's trip?

Answers

It takes him 3 days to finish

the ratio of the length to the breadth of a room is 5:3 what is the length of the room if the breadth is 6m

Answers

According to the solving If the room's width is 6 meter's, then its length must be 10 meter's.

What is length?

Distance is measured by length. The quantity of length has the dimension of distance there in International System of Quantities. Most measurement systems choose a base unit for length from which all subsequent units are derived. The metre serves as the fundamental unit of length in the International System of Units.

How do you determine length?

If you have an area A and a width w, you can calculate the length w by using the formula h = A/w. Its length can indeed be calculated using the formula h = P/2w if you know its perimeter (P) and width (W). If you have the width w and the diagonal

According to the give information:

Let length be x By Given Condition

5/3=x/6

x=5/3*6

x=30/3=10

Therefore length of room is 10.

To know more about length visit:

brainly.com/question/8552546

#SPJ1

Nathan and Lucy are selling blueberry and apple pies for a fundraiser. Nathan sold and 13 apple pies for a total of $147. Lucy sold 7 blueberry pies and 11 apple $204. What is the cost of one blueberry pie and one blackberry pie?

Answers

The cost of  one blueberry pie and one blackberry pie is $11.30 and $11.373 respectively.

What are mathematical operators?

In mathematics, an expression is a group of numbers and operations. The components of a mathematical expression that perform an operation are as follows:

Operands: The numbers used in an operation are called operands. Depending on the type of operation, different terms are applied to the operands.

Given Nathan sold 13 apples pies for $147,

cost of 13 apple pies  = $147

cost of 1 apple pie = 147/13 = $11.30

Lucy sold  7 blueberry pies and 11 apple pie $204.

7 blueberry  + 11 apple pie = $204

cost of 11 apple pie  = $124.384

cost of 7 blueberry = 204 - 124.384 = $79.615

cost of 1 blue berry = 79.615/7 = $11.373

Hence the cost of 1 apple pie is $11.30 and 1 blueberry is $11.373.

Learn more about Mathematical operators;

https://brainly.com/question/8959976

#SPJ1

-50 POINTS- If you guess will be reported.
If P=(1,6), Find: D3 (P)

Answers

Answer:

(3,18)

Step-by-step explanation:

You do not give the center of dilation, so I am using the origin (0,0)

You multiply the x and y by 3

(1,6)

1 x 3 = 3

(1,6)

6 x 3 = 18

(3,18)

A line passes through the point (-10,1) and has a slope of 3/2. Write an equation in point-slope form for this line.

Answers

The line passes through the point (-10,1) and has a slope of 3/20 has the equation as: y - 1 = 3/2 (x + 10)

How to write the equation of the line

The general form of a linear equation is y - y₁ = m (x - x₁).

It draws attention to the line's slope and one of the line's points (that is not the y-intercept)

The point slope formula is given by y - y₁ = m (x - x₁), where:

y = output value

y₁= the given y coordinate point

m = slope

x = input value

x₁ = the given x coordinate

substituting the point (-10,1) and slope 3/2

y - y₁ = m (x - x₁)

y - 1 = 3/2 (x + 10)

Learn more on linear equation at:

https://brainly.com/question/14323743

#SPJ1

Pls, solve this equation with the steps u took. 30 POINTS! posted this for the 2nd time

Answers

Answer:

x=2/3

Step-by-step explanation:

when the three turned into a 2 it removed one third, so we need to remove one third from 1, which gives us 2/3.

Basically, if a rectangle side goes from x to y, we figure out the percent decrease, and subtract that percent from the other side to find the variable we need to find on the other rectangle

n% of 28 is 14 `percentage`

Answers

Answer:

The Percentage is 50% because 14 is the half of 28.

Mike went on a bicycle trip. Every 4 days he travels 230 miles.
Create a proportional table comparing miles traveled to days spent traveling.

Answers

Calculating the ratio between each pair of values in a table allows you to determine whether a proportional relationship is present. The table depicts a proportional relationship if all of those ratios are the same.

How should an equation for a proportional table be written?

y = k x, where is the proportionality constant, is the equation that depicts a proportional relationship, or a line. Find k and create the equation by using k = y x from a table or graph. Tables, graphs, and equations can all be used to depict proportional relationships.

How should a proportional relationship be written?

The equation y = kx represents a proportional relationship between two quantities y and x that have the same proportionality constant, k.

To know more about proportional visit:-

brainly.com/question/29259009

#SPJ1

which is the highest 4,16,77 and lowest​

Answers

Answer:

highest 77 and lowest is 4

Please help. Assignment is due today. I am terrible with math, I don't understand this. Thanks in advance.

Answers

Answer:

a. A' (-3, -1) B' (3, 2) C' (6, 0); b. A' (-1, -3) B' (1, 6) C' (2, 0); c. A' (-2, -3) B' (2, 6) C' (2, 0)

Step-by-step explanation:

Solutions for graph a, b and c are attached.

a. (x, y) to (3x, y)

A' (-3, -1) B' (3, 2) C' (6, 0)

b. (x, y) to (x, 3y)

A' (-1, -3) B' (1, 6) C' (2, 0)

c. (x, y) to (2x, 3y)

A' (-2, -3) B' (2, 6) C' (2, 0)

REMEMBER: Connect the lines to make the triangles for each graph BEFORE submitting.

evaluate 7 to the 3rd power x 7 to the 3rd power

Answers

Answer:117649

Step-by-step explanation:

Write the expression as a decimal number 2 x 0.1 + 5 x 0.001 =

Answers

The measure of decimal number is 2 tenths + 5 thousandths is 0.205.

What is expression?

Mathematical expressions consist of at least two numbers or variables, at least one arithmetic operation, and a statement. It's possible to multiply, divide, add, or subtract with this mathematical operation. An expression's structure is as follows:

Expression: (Math Operator, Number/Variable, Math Operator)

Given expression

2 x 0.1 + 5 x 0.001

the expanded form is 2 tenths + 5 thousandths

= 2/10 + 5/1000

multiply and divide by 100 in 2/10

we get 200/1000

now 200/1000 + 5/1000 = 205/1000 = 0.205

Hence value of expression is 0.205.

Learn more about expression;

https://brainly.com/question/14083225

#SPJ1

-50 POINTS- don’t explain

Answers

i am pretty sure it is 3..just due to the structure of the triangle. BUT DONT TAKE MY WORD FOR IT. but i am almost positive it is 3

Answer:

3

Step-by-step explanation:

like you said don't explain

I need help, I know it gave me the answer but I don’t understand how it got (-3) someone explain please

Answers

Answer: h=14

Step-by-step explanation:

If x=-18, then

h=17+(-18)/6

h=17+(-3)

h=17-3

h=14

Answer:

Step-by-step explanation:

so, the calculation was :  h = 17 + [tex]\frac{-18}{6\\}[/tex]

To subtract "[tex]\frac{-18}{6}[/tex] " we need to make it the same denominator  as "17"

(it's not written but technically 17 = [tex]\frac{17}{1}[/tex] )

Yet, we see that [tex]\frac{-18}{6} =\frac{-3* 6}{1*6}[/tex]

We just need to remove both the 6 that are not useful here and :

[tex]\frac{-3*6}{1*6} =\frac{-3}{1}= -3[/tex]

We obtain h = 17 - 3 = 14

hope it helped :)

-POINTS-
(4IZ0 Don’t even try.)

Answers

Answer:

Step-by-step explanation:

axf

I don’t know what to put here

Sue ate one sweetie one day, and each afterward she ate one sweetie more than she did the day before. How many sweeties did she eat during her first week?​

Answers

on solving the provided question we can say that - by unitary method we have 12 days.

What is unitary method ?

The unit technique is an approach to problem-solving that involves first determining the value of a single unit, then multiplying that value to determine the required value. The unit method, to put it simply, is used to extract a single unit value from a supplied multiple. For instance, 40 pens would cost 400 rupees, or the price of one pen. The process for doing this may be standardized. a single country. anything that has an identity element. (mathematics, algebra) (Linear algebra, mathematical analysis, mathematics of matrices or operators) Its adjoint and reciprocal are equivalent.

Sue ate one sweetie one day,

Sue ate 12 sweetie 12 day,

To know more about unitary method  visit:

https://brainly.com/question/28276953

#SPJ1

Other Questions
if AB||EF and EF||CD, then find the value of x. 1. A student has computed that it takes an average of 17 minutes with a standard deviation of 3 minutes to drive from home, park the car, and walk to an early morning class. Find the proportion of times whose time X satisfies each of the following conditions:A) If you selected 10 days at random, what is the probability that half the days will take between 15 and20 minutes?B) What is the probability the second day selected will be the first day he is late to class, assuming he leftat 8:39am?(I know how to solve for the probabilities on their own, however Im not sure how to incorporate the added probability based on multiple days) Jonathan opened an auto repair shop. In the first month, 11 cars were serviced at his shop. After the first month, the total number of cars serviced at Jonathan's shop increased by 9 cars per month.How many total cars were serviced at Jonathan's shop at the end of the 7th month? What you give up in the future to make a purchase today? The nurse is preparing a teaching plan for a pregnant woman about the signs and symptoms to be reported immediately to her health care provider. Which signs and symptoms would the nurse include The health care provider has just prescribed tetracycline for an adolescent with acne vulgaris. The client takes oral contraceptive pills. The clinic nurse should educate the teen about which topics? Select all that apply.1. Not taking tetracycline with dairy products2. Taking tetracycline at bedtime3. Taking tetracycline with food4. Using additional contraceptive techniques5. Using sunblock regard placenta previa which of the following is true:1. placenta previa can't be diagnosed by ultrasound2. majority of low lying placenta at 20 week will remain so at term 3. the patient should be routinely managed as in patient with delay of delivery until the onset of labor 4. complication of placenta previa include need for cesarean, bleeding, accrete The type of bonding and the number of covalent bonds an atom can form with other atoms is determined by __________ 560Steve can paint his greenhouse in four hours working alone; his ten-year-old son can paint the greenhouse alone in nine hours. Working together, how long, to the nearest minute, would it take them Please help - How many whole months will it take for a motorbike valued at 2550 to depreciate to less than 1200 if depreciation is at a rate of 5% per month? Match the phase described Can you incorporate open-source code from a github forum into IP tool? Which absolute value function, when graphed, will bewider than the graph of the parent function, f(x) = |x|? helpppppppppppppppppppppppppppppp The width of a rectangle is represented by 5k4h 6r; the length of the rectangle is represented by 7k4h 6r. Which of the following expressions represents the perimeter of the rectangle 25 Points! Hurry! Please help! I provided a screenshot of my question. Thank you! The uniform beam has a weight W and length L and is supported by a pin at A and cable BC ( figure 1)Part ADetermine the horizontal and vertical components of reaction at AExpress your answers in terms of some or all of the variables W, R. and ). Enter your answers separated by a comma.View Available Hint's)Aa + Ay = ____Parts B Determine the one cable car necessary to hold the beans position shownExpress your answers in terms of some or all of the variables W, R. and ). A bank account is opened in the name of 'X' with full KYC of 'X'. 'X' later sells his account to 'Y', who is now using the account and 'X' is not aware of the trasnactions in the account. Such an account is called_ What happens if a cell gets too large for the amount of DNA it has? How long can killer whales go without air?